LSAT and Law School Admissions Forum

Get expert LSAT preparation and law school admissions advice from PowerScore Test Preparation.

 Administrator
PowerScore Staff
  • PowerScore Staff
  • Posts: 8916
  • Joined: Feb 02, 2011
|
#27347
Complete Question Explanation

Strengthen—PR, CE. The correct answer choice is (C)

In this stimulus, the writer draws a causal link between collaboration and a book’s success:

  • Cause ..... ..... :arrow: ..... ..... Effect

    collaboration ..... ..... the book’s success

Based on this causal premise, the writer concludes that there should be collaboration on the next book so that it too has good sales. The question stem asks that we find the principle which most strengthens this conclusion. The correct answer choice will probably prescribe a given approach based on past success.

Answer choice (A): There is no mention in the stimulus that the writer is contemplating a collaboration with the same author as last time, so this answer choice fails to support the writer’s conclusion.

Answer choice (B): This conditional statement, which assert that success requires collaboration, does not support the author’s conclusion that collaboration is advisable because it brought success last time (not because it is absolutely required to succeed, but that collaboration will make success more likely).

Answer choice (C): This is the correct answer choice. If this principle is true, then the writer’s argument becomes stronger. Based on the causal link between collaboration and success, this choice would suggest that future collaborations will enjoy similar success.

Answer choice (D): The writer in the stimulus does not assert that collaborators always have a better chance of selling well, but that the past successful collaboration means that for this writer, future collaboration is advisable as well.

Answer choice (E): This answer choice fails to lend support because it provides an alternative cause for a book’s success—a collaboration of good writers. The writer in the stimulus does not profess to be a good writer or to have collaborated with a good writer. The claim in the stimulus is simply this: collaboration led to this writer’s successful sales of the last book, so the writer should collaborate again in order to achieve good sales on the next book.
User avatar
 DaishiMT
  • Posts: 4
  • Joined: Jan 10, 2022
|
#93824
I thought B is the strongest answer because it is ensuring the no other reason, aside from collaborating on the book with another writer, can lead to a book selling well. The answer bolsters the Cause and Effect of collaboration :arrow: the book's success. Can someone kindly explain why this isn't the right answer?
It is because the author's conclusion isn't the causal link but rather that they "should collaborate with a writer on my next book so that book will sell well too"? Thanks!
 Adam Tyson
PowerScore Staff
  • PowerScore Staff
  • Posts: 5153
  • Joined: Apr 14, 2011
|
#93839
Answer B has two problems, DaishiMT, and the first is that it is purely conditional while the argument is primarily causal (the book sold well because of the collaboration), The second, though, is the real killer, and it is that the answer has the relationship backwards, conditionally speaking.

If we ignore the causal reasoning for a moment, the author is arguing that IF they collaborate, THEN the book will sell well. Answer B is saying IF a book sells well, THEN the author must have collaborated. It's a Mistaken Reversal of the author's argument! Just because a book requires collaboration in order to sell well does not mean that collaboration will cause that book to sell well. Thus, answer B really doesn't help the claim that a collaboration will cause the book to sell well. Even if B is true, there could still be many books that do not sell well even though they involved writers collaborating!
User avatar
 emilyjmyer
  • Posts: 48
  • Joined: May 11, 2022
|
#95508
Hi,

For this question the stem is "Which one of the following principles, if valid, most helps to justify the reasoning above?"

How do we know this is a strengthen stem and nota justify the conclusion stem?

Thanks!
 Robert Carroll
PowerScore Staff
  • PowerScore Staff
  • Posts: 1787
  • Joined: Dec 06, 2013
|
#95549
Emily,

Dave's post here is relevant: viewtopic.php?p=9480#p9480

And I will specifically quote that part of his post that addresses your question:

But, the following aren't Justify questions, but are instead Strengthen questions:
Which one of the following, if valid, most helps to justify the reasoning above?
Which one of the following, if assumed, does most to justify the argument’s conclusion?
Why? Because the most in each means that 100% soundness isn't required, and since a Justify question is 100%, these don't qualify and are instead Strengthen questions.

Robert Carroll
User avatar
 sunshine123
  • Posts: 44
  • Joined: Jul 18, 2022
|
#97142
Hello,

I understand the question but I have a more general question about principal type questions. When we are looking for a principle that justifies a certain set of reasoning, the principle, first of all, better embody the conclusion of the reasoning; that is , the principle better take you to the conclusion. However, as is often the case, like in this problem, the principle ought to also embody some of the premises of the reasoning, the reasoning of the reasoning. Is this always the case without fail?

Best,
Dylan Ramirez
 Adam Tyson
PowerScore Staff
  • PowerScore Staff
  • Posts: 5153
  • Joined: Apr 14, 2011
|
#97177
I think that's a fair assessment, Dylan. I suppose it's possible to justify a conclusion without reference to the premises, but on the LSAT we are always (almost always?) looking to definitively tie the premises to the conclusion, so the principle should connect something from the premises to the key idea in the conclusion. A simple formular that may help is:

"If (the premises), then (the conclusion)."

Get the most out of your LSAT Prep Plus subscription.

Analyze and track your performance with our Testing and Analytics Package.